Please confirm topic selection

Are you sure you want to trigger topic in your Anconeus AI algorithm?

Please confirm action

You are done for today with this topic.

Would you like to start learning session with this topic items scheduled for future?

Review Question - QID 3776

QID 3776 (Type "3776" in App Search)
A 60-year-old female presents with a 2-month history of low back pain. Imaging is seen in Figures A and B. Which of the following is LEAST likely to be true?
  • A
  • B
  • A
  • B